From fcf4d85007dcfcbdc6220331f5f9d9ba31f6ae67 Mon Sep 17 00:00:00 2001 From: Yigit Sever Date: Sat, 7 Nov 2020 04:27:52 +0300 Subject: Last part of the question Only last part of 4 left --- main.tex | 51 +++++++++++++++++++++++++++++++++++---------------- 1 file changed, 35 insertions(+), 16 deletions(-) diff --git a/main.tex b/main.tex index 8002998..fe20d1f 100644 --- a/main.tex +++ b/main.tex @@ -552,12 +552,12 @@ For the right hand side; we can start by picking $n_{0} = 1$ and $c = 1$ in the In order to prove the right hand side of the inequality; we can pick $c_2 > 2$ which equals; -\begin{eqnarray*}% +\begin{align*}% \label{eq:c2gt2} - f(n) + g(n) \le 2 \cdot f(n) \\ - \cancel{f(n)} + g(n) \le \cancel{2 \cdot} f(n) \\ - g(n) \le f(n) -\end{eqnarray*} + f(n) + g(n) &\le 2 \cdot f(n) \\ + \cancel{f(n)} + g(n) &\le \cancel{2 \cdot} f(n) \\ + g(n) &\le f(n) +\end{align*} Which we proved above using Equation~\ref{eq:a_little_o}. This conjecture is \emph{true}. @@ -589,8 +589,7 @@ Which we proved above using Equation~\ref{eq:a_little_o}. This conjecture is \em Hence $f(n) = \mathcal{O}(n^{2})$ and $H=2$. -% TODO: reread proof, strengthen? <06-11-20, yigit> % -(2)$L=2$. We can pick the constant $c = \frac{1}{2}$ and write the Big-$\Omega$ notation; +(2) $L=2$. We can pick the constant $c = \frac{1}{2}$ and write the Big-$\Omega$ notation; \begin{gather*} c > 0 ~ \text{and} ~ n_{0} \ge 0 \\ @@ -601,21 +600,41 @@ Hence $f(n) = \mathcal{O}(n^{2})$ and $H=2$. Does a $L>2$ work? Pick any $\ell > 2$; -\begin{gather*} - c > 0 ~ \text{and} ~ n_{0} \ge 0 \\ - \frac{n(n+1)}{2} \ge \frac{1}{2} n^{\ell} \\ - \frac{n(n+1)}{\cancel{2}} \ge \frac{1}{\cancel{2}} n^{\ell} \\ - n(n+1) \ge n^{\ell} \\ - n+1 \ge n^{\ell - 1} \\ - 1 \ge n(n^{\ell - 2} - 1) -\end{gather*} +\begin{align*} + c > 0 ~ \text{and} ~ n_{0} &\ge 0 \\ + \frac{n(n+1)}{2} &\ge \frac{1}{2} n^{\ell} \\ + \frac{n(n+1)}{\cancel{2}} &\ge \frac{1}{\cancel{2}} n^{\ell} \\ + n(n+1) &\ge n^{\ell} \\ + n+1 &\ge n^{\ell - 1} \\ + 1 &\ge n(n^{\ell - 2} - 1) +\end{align*} -The final equation does not hold $n>1$ so $L>2$ cannot be true. +The final equation does not hold for $n>1$ or the assumed $\ell > 2$ so $L>2$ cannot be true. \begin{info}[] $f(n) = \sum^{\lceil{\log{n}}\rceil}_{k=0} \frac{n}{2^{k}}$ \end{info} +It's beneficial to unpack the infinite sum beforehand; + +\begin{align*} + f(n) &= \sum^{\lceil{\log{n}}\rceil}_{k=0} \frac{n}{2^{k}}\\ + &= \frac{n}{2^{0}} + \frac{n}{2^{1}} + \frac{n}{2^{2}} + \dots + \frac{n}{2^{\lceil{\log{n}}\rceil}} \\ + &= n \Big(1 + \frac{1}{2^{1}} + \frac{1}{2^{2}} + \dots + \frac{1}{2^{\lceil{\log{n}}\rceil}} \Big) \\ +\end{align*} + +The infinite sum approaches 1 as $\lceil{\log n\rceil} \rightarrow \infty$, giving us; + +\begin{equation*} + f(n) = 2 \cdot n \\ +\end{equation*} + +Finally, $H = L = 1$ since $f(n)$ is $\Theta(n)$. Borrowing the Big-$\Theta$ notation from Equation~\ref{eq:a_big_theta} for $c_1 = 1$ and $c_2 = 2$; + +\begin{equation*} + n \le f(n) \le 2n +\end{equation*} + \begin{info}[] $f(n) = n(\log n)^{2}$ \end{info} -- cgit v1.2.3-61-g4310